How Does U-Substitution Using Tan(x/2) Simplify Trigonometric Integrals?

  • Thread starter Thread starter MrOranges
  • Start date Start date
Click For Summary
Using the substitution u = tan(x/2) simplifies trigonometric integrals by allowing the transformation of sin(x) and cos(x) into rational functions of u. Specifically, sin(x) becomes 2u/(u^2 + 1) and cos(x) becomes (1 - u^2)/(u^2 + 1). This approach leverages the identity 1 + tan^2(α) = 1/cos^2(α) to derive these relationships. Additionally, constructing a right triangle based on u = tan(x/2) can provide a visual method to find sin(x/2) and cos(x/2). Ultimately, this substitution facilitates the evaluation of integrals like sin(x)/(sin(x) + cos(x)).
MrOranges
Messages
3
Reaction score
0

Homework Statement



Im trying to figure out how to get sinx=2u/(u^2+1) and cosx=(1-u^2)/(u^2+1). In order to solve the problem integral sinx/(sinx+cosx)

Homework Equations


use tan(x/2) to solve the problem.

The Attempt at a Solution


all I can get to is u=sin(x/2)/cos(x/2)-------2u=sinx/cosx
I don't understand how we can get the u^2+1 on the bottom. All we need is the give the u-substitution but i donno how to do it.
 
Last edited:
Physics news on Phys.org
So, we let u=\tan(x/2). Then we need to evaluate

\frac{2u}{1+u^2}=\frac{2\tan(x/2)}{1+\tan^2(x/2)}

Try to prove the following formula first:

1+\tan^2(\alpha)=\frac{1}{\cos^2(\alpha)}
 
I just want to know how you get sinx=2u/(u^2+1) and cosx=(1-u^2)/(u^2+1) from tan(x/2). After that I can use that to plug in the integral sinx/(sinx+cosx) to make the equation solvable.
 
I don't quite get it. Just substitute u=\tan(x/2) and you'll see that the equation is correct...
 
thanks, i got it now
 
Hello,

Another easier approach :

You have u=tan(x/2), draw the triangle.

Now, sin(x)=sin( 2 (x/2) ) = 2 sin(x/2) cos(x/2) , get sin(x/2) & cos(x/2) from your triangle.
cos(x)=cos( 2 (x/2) ) = [ cos(x/2) ]^2 - [ sin(x/2) ]^2
 
Question: A clock's minute hand has length 4 and its hour hand has length 3. What is the distance between the tips at the moment when it is increasing most rapidly?(Putnam Exam Question) Answer: Making assumption that both the hands moves at constant angular velocities, the answer is ## \sqrt{7} .## But don't you think this assumption is somewhat doubtful and wrong?

Similar threads

Replies
3
Views
2K
Replies
3
Views
2K
  • · Replies 6 ·
Replies
6
Views
2K
  • · Replies 3 ·
Replies
3
Views
1K
  • · Replies 12 ·
Replies
12
Views
2K
  • · Replies 3 ·
Replies
3
Views
3K
  • · Replies 8 ·
Replies
8
Views
2K
  • · Replies 2 ·
Replies
2
Views
3K
  • · Replies 16 ·
Replies
16
Views
2K
  • · Replies 10 ·
Replies
10
Views
2K